Sie sind auf Seite 1von 49

PLANCESS RANK ACCELERATOR

PHYSICS
FOR JEE MAIN & ADVANCED

Moving Charges and Magnetism

4000+questions
with topic wise exercises

2000+ problems
of IIT-JEE & AIEEE exams
of last 35 years

4 Levels of
Exercises categorized
into JEE Main & Advanced

7 Types of Questions
based on latest JEE pattern

Detailed Solutions
of all questions are available PlancEssential
Questions recommended
for revision

www.plancess.com
Moving Charges and Magnetism

EXERCISE 1 JEE MAIN/BOARDS


Q.1 A circular coil of wire consisting of 100 anticlockwise, and clockwise in Y, for an
turns, each of radius 8.0 cm carries current of observer looking at the coils facing west. Give
0.40 A. What is the magnitude of the magnetic the magnitude and direction of the net
field B at the center of the coil? magnetic field due to the coils at their center.

Q.2 A long straight wire carries a current of 35 Q.9 A straight horizontal conducting rod of
A. What is the magnitude of the field B at a length 0.45 m and mass 60 g is suspended by
point 20 cm from the wire? two vertical wires at its ends. A current of 5.0 A
is set up in the rod through the wires.
Q.3 A long straight wire in the horizontal plane
(a) What magnetic field should be set up
carrier of 50 A in north to south direction. Give
normal to the conductor in order that the
the magnitude and direction of Bat a point 2.5
tension in the wire is zero?
m east of the wire.
(b) What will be the total tension in the wires if
Q.4 A horizontal overhead power line carries a the direction of current is reversed keeping the
current of 90 A in east west direction. What is magnetic field same as before?
the magnitude and direction of the magnetic
Q.10 The wires which connect the battery of an
field due to the current 1.5 m below the line?
automobile to its starting motor carry a current
Q.5 What is the magnitude of a magnetic force of 300 A (for a short time). What is the force per
per unit length on a wire carrying a current of 8 unit length between its wires if they are 70 cm
A and making an angle of 30 with the long and 1.5 cm apart? Is the force attractive of
direction of a uniform magnetic field of 0.15 T? repulsive?

Q.6 In a chamber, a uniform magnetic field of Q.11 A uniform magnetic field of 1.5 T exists in
6.5 G(1G= 104 T) is maintained. An electron is a cylindrical region of radius 10.0 cm, its
shot into the field with a speed of 4.8  106 direction parallel to the axis along east to west.
ms1 normal to the field. Explain why the path A wire carrying current of 7.0 A in the north to
of the electron is a circle. Determine the radius south direction passes through this region.
of the circular orbit. What is the magnitude and direction of the

e  1.6  10 19
C, me  9.1  10 31 kg  force on the wire if,
(a) the wire intersects the axis,
Q.7 (a) A circular coil of 30 turns and radius 8.0 (b) the wire is turned from N-S to northeast-
cm carrying a current of 6.0 A is suspended northwest direction,
vertically in a uniform horizontal magnetic field (c) the wire in the N-S direction is lowered from
of magnitude 1.0 T. The field lines make an the axis by a distance of 6.0 cm?
angle of 60 with the normal of the coil. Q.12 A circular coil of N turns and radius R
Calculate the magnitude of the counter torque carries a current I. It is unwound and rewound
that must be applied to prevent the coil from to make another coil of radius R/2. Current I
turning. remaining the same. Calculate the ratio of the
(b) Would your answer change, if the circular magnetic moments of the new coil and the
coil in (a) were replaced by a planner coil of original coil.
some irregular shape that encloses the same
area? (All other particulars are also unaltered.) Q.13 A circular coil of 20 turns and radius 10 cm
is placed in a uniform magnetic field of 0.10 T
Q.8 Two concentric circular coils X and Y radii normal to the plane of the coil. If the current in
16 cm and 10 cm, respectively, lie in the same the coil is 5.0 A, what is the
vertical plane containing the north to south (a) total torque on the coil,
direction. Coil X has 20 turns and carries a (b) total force on the coil
current 16 A; coil Y has 25 turns and carries a (c) average force on each electron is the coil
current of 18 A. The sense of the current in X is due to the magnetic field?

21.1
www.plancess.com
Moving Charges and Magnetism

(The coil is made of copper wire of cross- the magnetic field at its center? Use the
sectional area 105 m2 , and the free electron standard value of  .
0
density in copper is given to be about 1029 m3 .)
Q.24 A proton enters a magnetic field of flux
Q.14 State the Biot-Savart law for the magnetic
density 2.5 T with a velocity of 1.5  107 ms1 at
field due to a current-carrying element. Use this
an angle of 30 with the field. Find the force on
law to obtain a formula for magnetic field at the
the proton.
center of a circular loop of radius a carrying a,
steady current I. Q.25 Two parallel wires one meter apart carry
currents of 1A and 3 respectively in opposite
Q.15 Give the formula for the magnetic field
directions. Calculate the force per unit length
produced by a straight infinitely long current-
acting between these wires.
carrying wire. Describe the lines of field B in this
case. Q.26 A solenoid of length 0.4, and having 400
turns of wire carries a current of 3 A. A thin coil
Q.16 How much is the density B at the center of
having 10 turns of wire and radius 0.01 m
a long solenoid?
carries a current 0.4 A. Calculate the torque
Q.17 A proton shot at normal to magnetic field required to hold the coil in the middle of the
describe a circular path of radius R. If a solenoid with its axis perpendicular to the axis

  2
deuteron 1 H is to move on the same path,
of the solenoid.

Q.27 In a circuit
what should be the ratio of the velocity of
shown in Fug.
proton and the velocity of deuteron?
21.82 a voltmeter
Q.18 State the principle of cyclotron. reads 30 V, when
it is connected
Q.19 A charge q is moving in a region where
across 400ohm
both the magnetic field B and electric field E are
resistance. Figure 21.82
simultaneously present. What is the Lorentz
Calculate what
force acting on the charge?
the same voltmeter will read when connected
Q.20 A charged particle moving in a straight across the 300  resistance?
line enters a uniform magnetic field at an angle
Q.28 Two long straight parallel wires are 2m
of 45 . What will be its path?
apart, perpendicular to the plane of the paper.
Q.21 A current of 1A is The wire A carries a current of 9.6 ampere
flowing in the sides of an directed into the plane of the paper. The wire B
equilateral triangle of side carries a current such that the magnetic field
4.5  102 m. Find the 10
induction at the point P, at a distance of m
magnetic field at the 11
centroid of the triangle. from the wire B, is zero. Calculate
Figure 21.81 (i) the magnitude and direction of current in B
Q.22 The radius of the first electron orbit of a (ii) the magnitude of magnetic field induction
hydrogen atom is 0.5 Å. The electron moves in at S
this orbit with a uniform speed of (iii) the force per unit length of the wire B.
2.2  106 ms1 . What is the magnetic field
produced at the center of the nucleus due to
the motion of this electron?

Q.23 A solenoid is 2 m long and 3 cm in


diameter. It has 5 layers of windings of 1000
turns each and carries a current of 5 A. What is

21.2
www.plancess.com
Moving Charges and Magnetism

EXERCISE 2 JEE MAIN


Single Correct Question 0 I
(A) 3 (B) zero
Q.1 A current 1 ampere is 2R
flowing through each of
the bent wires as shown.
(C)  2 1  2RI0
(D)  3 2  2RI
0

The magnitude and


direction of magnetic field Q.5 Two concentric coils X and Y of radii 16 cm
at O is Figure 21.83 and 10 cm lie in the same vertical plane
containing N-S direction. X has 20 turns and
0i  1 2 
(A)    (B) carries 16 A. Y has 25 turns & carries 18 A. X has
4  R R  current in anticlockwise direction and Y has
0i  1 3  current in clockwise direction for an observer,
   looking at the coils facing the west. The
4  R R 
magnitude of net magnetic field at their
0i  1 3  0i  1 3 
common center is
(C)    (D)   
8  R 2R  8  R R  (A) 5  104 T towards west
4
Q.2 Net magnetic field at the (B) 13 10 T towards east
center of the circle O due to a (C) 13 10
4
T towards west
current carrying loop as
4
shown in Fig. 21.84 is (D) 5 10 T towards east
   180 Figure 21.84 Q.6 Equal current i is flowing in three infinitely
(A) zero long wires along positive x, y and z directions.
(B) perpendicular to paper inwards The magnetic field at a point (0, 0, -a) would be:
(C) perpendicular to paper outwards 0i ˆ ˆ 0i ˆ ˆ
(D) is perpendicular to paper inwards if  
(A)
2a
j i   (B)
2a

i j 
90 and perpendicular to paper outwards if  i  i
90    180
(C) 0 ˆi  ˆj
2a
  
(D) 0 ˆi  ˆj  kˆ
2a

Q.3 A charge particle A of Q.7 An electron is moving along positive x-axis.
charge q=2C has velocity A uniform electric field exists towards
v=100 m/s. When it passes negatively y-axis. What should be the direction
through point A and has of magnetic field of suitable magnitude so that
velocity in the direction net force of electron is zero.
shown. The strength of Figure 21.85 (A) Positive z-axis (B) Negative z-axis
magnetic field at point B due (C) Positive y-axis (D) Negative y-axis
to this moving charge is (r=2 m).
(A) 2.5 T (B) 5.0 T Q.8 A particle of charge q and mass m starts
moving from the origin under the action of an
(C) 2.0 T (D) None
electric field E  E0ˆi and B  B0ˆi with velocity
Q.4 Three rings, each
v  v 0ˆj . The speed of the particle will become
having equal radius R, are
placed mutually 2 v 0 after a time
perpendicular to each 2mv 0
other and each having its (A) t  (B) t  2Bq
qE mv 0
center at the origin of co-
ordinates system. If current Figure 21.86 3Bq 3 mv 0
(C) t  (D) t 
is flowing through each mv 0 qE
ring then the magnitude of the magnetic field
at the common center is

21.3
www.plancess.com
Moving Charges and Magnetism

Q.9 An electron is projected with velocity v 0 in 3R 3


(A) (B) R
a uniform electric field E perpendicular to the 2 2
field. Again it is projected with velocity v 0 2 4
(C) R (D) R
perpendicular to a uniform magnetic field B. If 9 3
r1 is initial radius of curvature just after entering Q.14 A charged particle moves in magnetic
in the electric field and r2 in initial radius of field B  10 i with initial velocity u  5i  4 j .
curvature just after entering in magnetic field The path of the particle will be.
then the ratio r1 r2 is equal to (A) Straight line (B) Circle
(C) Helical (D) None
2 B
(A) Bv 0 (B)
E E Q.15 A electron experiences a force
(C)
Ev 0
B
(D)
Bv 0
E
 
4.0iˆ  3.0ˆj x10 13 N in a uniform magnetic
7 1
field when its velocity is 2.5k  10 ms .
Q.10 A uniform magnetic field B  B0ˆj exists in
When the velocity is redirected and becomes
a space. A particle of mass m and charge q is
projected towards negative x-axis with speed v
 
1.5iˆ  2.0ˆj x107 ms 1 , the magnetic force of

from the point (d, 0, 0). The maximum value v the electron is zero. The magnetic field vector B
for which the particle does not hit y-z plane is is :
2Bq Bqd (A) 0.075i  0.1 j (B) 0.1i  0.075 j
(A) (B)
dm m
Bq Bqd (C) 0.075i  0.1 j  k (D) 0.075i  0.1 j
(C) (D)
2dm 2m
Q.16 An electron moving with a velocity
Q.11 Two protons move parallel to each other, ˆ / s at a point in a magnetic field
V1  2im
keeping distance r between them, both moving
with same velocity V. Then the ratio of the experiences a force F  2jˆ N . If the electron is
1
electric and magnetic force of interaction moving with a velocity V2  2 jm / s at the same
between them is.
2 2 2 2
point, it experiences a force F   2iˆ N . The
(A) c / V (B) 2c / V 2

force the electron would experience if it were


2 2
(C) c / 2V (D) None ˆ / s at the
moving with a velocity V3  2km
  same point is
Q.12 Three ions H ,He and O2 having same
(A) Zero (B) 2kˆ N
kinetic energy pass through a region in which
there width is a uniform magnetic field (C) 2kˆ N (D) Information is insufficient
perpendicular to their velocity, then:
Q.17 The direction of
(A) H will be least deflected.
magnetic force on the
(B) He and O2 will be deflected equally.
electron as shown in the
(C) O2 will be deflected most.
diagram is along
(D) all will be deflected equally. (A) y-axis
Q.13 An electron having kinetic energy T is (B) –y-axis
moving in a circular orbit of radius R (C) z-axis
perpendicular to a uniform magnetic induction (D) –z-axis Figure 21.87
B. If kinetic energy is doubled and magnetic
induction tripled, the radius will become.

21.4
www.plancess.com
Moving Charges and Magnetism

Q.18 A block of mass m & charge q is released of a wire, C is a capacitance and R is a


on a long smooth inclined plane magnetic field resistance. All other symbols have standard
B is constant, uniform, horizontal and parallel meanings.
to surface as shown. Find the time from start (A) x, y have the same
when block loses contact with the surface. dimensions
(A) mcos  (B) mcosec  (B) y, z have the same
qB qB dimension
(C) mcot  (D) None (C) z, x have the same
qB dimensions
(D) None of the three Figure 21.89
Q.19 A metal ring of radius r=0.5m with its pairs have the same
plane normal to a uniform magnetic field B of dimensions.
induction 0.2T carries a current I=100A. The
tension in Newtons developed in the ring is: Q.24 Two long thin, parallel conductors
(A) 100 (B) 50 carrying equal currents in the same direction
(C) 25 (D) 10 are fixed parallel to the x-axis, one passing
through y=a and the other through y=-a. The
Q.20 In the Fig. 21.88 shown a coil of single turn resultant magnetic field due to the two
is wound on a sphere of radius R and mass m. conductors at any point is B. Which of the
The plane of the coil following are correct?
is parallel to the (A) B=0 for all points on the x-axis
plane and lies in the (B) At all points on the y-axis, excluding the
equatorial plane of origin, B has
the sphere. Current in only a z-
the coil is i. The value Figure 21.88 component.
of B if the sphere is in (C) At all
equilibrium is points on
mgcos  mg the z-axis,
(A) (B)
iR iR excluding
mg tan  mgsin  the origin, B
(C) (D)
iR iR Figure 21.90
has only an
Q.21 The magnetic moment of a circular orbit x-
of radius ‘r’ carrying a charge ‘q’ and rotating component.
with velocity v is given by Q.25 An electron is moving along the positive
qvr qvr X-axis. You want to apply a magnetic field for a
(A) (B)
2 2 short time so that the electron may reverse its
2 direction and move parallel to the negative X-
(C) qvr (D) qvr
axis. This can be done by applying the magnetic
MULTIPLE CORRECT QUESTION field along.
Q.22 A long straight wire carriers a current (A) Y-axis (B) Z-axis
along the x-axis. Consider the points A(0,1,0), (C) Y-axis only (D) Z-axis only
B(0,1,1), C(1,0,1) and D(1,1,1). Which of the
following pairs of points will have magnetic Q.26 Two identical charged particles enter a
field of the same magnitude? uniform magnetic field with same speed but at
(A) A and B (B) A and C angles 300 and 600 with field. Let a, b and c be
(C) B and C (D) B and D the ratio of their time periods, radii and pitches
of the helical paths then
Q.23 Consider three quantities x=E/B, (A) abc=1 (B) abc > 1
1 (C) abc < 1 (D) a=bc
y  1 / 00 and z  . Here, l is the length
CR

21.5
www.plancess.com
Moving Charges and Magnetism

Q.27 Consider the following statements Statement-2: The magnetic field due to finite
regarding a charged particle in a magnetic field. length of a straight current carrying wire is
Which of the statement are true : symmetric about the wire.
(A) Starting with zero velocity, it accelerates in
Q.32 Statement-1: A pendulum made of a non-
a direction perpendicular to the magnetic field.
(B) While deflecting in magnetic field its energy conducting rigid massless rod of length is
gradually increases. attached to a small sphere of a mass m and
(C) Only the component of magnetic field charge q. The pendulum is undergoing
perpendicular to the direction of motion oscillations of small amplitude having time
of the charged particle is effective in deflecting period T. Now a uniform horizontal magnetic
it. field out of plane of page is switched on. As a
(D) Direction of deflecting force on the moving result of this change, the time period of
charged particle is perpendicular to its velocity. oscillations will change.

ASSERTION REASON QUESTION


(A) Statement-1 is true, statement-2 is true and
statement-2 is correct explanation for
statement-1.
(B) Statement-1 is true, statement-2 is true and
statement-2 is NOT the correct explanation Figure 21.91
for statement-1.
(C) Statement-1 is true, statement-2 is false. Statement-2: In the situation of statement-1,
(D) Statement-1 is false, statement-2 is true. after the magnetic field is switched on the
tension in string will change (except when the
Q.28 Statement-1 : It is not possible for a
bob is at extreme position).
charged particle to move in a circular path
around a long straight uncharged conductor COMPREHENSION TYPE QUESTION
carrying current under the influence of its Comprehension-1
magnetic field alone. Magnetic field intensity (B) due to current
Statement-2: The magnetic force (if nonzero) carrying conductor can be calculated by use of
on a moving charged particle is normal to its 0 Idlxr
Biot-Savart law. Which is B  , where
velocity. 4 r3
dB is magnetic field due current element Idl at
Q.29 statement-1: For a charged particle to
a position r from current element. For straight
pass through a uniform electro-magnetic field
wire carrying current magnetic field at a
without change in velocity, its velocity vector
0 I
must be perpendicular to the magnetic field. distance R from wire is B 
4 R
 sin   sin 
Statement-2: Net Lorentz force on the particle
And magnetic field due to a circular arc at its
is given by F  q E  v xB center is
0 I
Q.30 statement-1: Two long parallel conductors B .
4 R
carrying current in the same direction
where  angle
experience a force of attraction.
of circular arc
Statement-2: The magnetic fields produced in
at center, R is
the space between the conductors are in the
radius of
same direction. Figure 21.92
circular arc.
Q.31 statement-1: Ampere law can be used to Now answer
find magnetic field due to finite length of a the following questions.
straight current carrying wire.
Q.33 The magnetic field at C due to curved part
is

21.6
www.plancess.com
Moving Charges and Magnetism

0 I The coil is placed in a uniform magnetic field of


(A) , directed into the plane of the paper
6 0.5 Wbm-2. Then, the dipole moment of the coil
0 I is
(B) , directed towards you
6 (A) 3000Am2 (B) 0.3Am2
0 I 2
(C) , directed towards you (C) 75Am (D) 1.88x102 Am2
3
0 I Q.37 A current of 3A is flowing in a plane
(D) , directed up the plane of the paper
3 circular coil of radius 1cm and having 20 turns.
The coil is placed in a uniform magnetic field of
Q.34 A wire loop
0.5 Wbm-2. Then, the P.E. of the magnetic
carrying a current I is
dipole in the position of stable equilibrium is
shown in Fig. 21.93.
(A) -1500 J (B) -9.4 mJ
The magnetic field
(C) +0.15 J (D) +1500 J
induction at C due to
straight part is Q.38 In above question, to hold the current-
carrying coil with the normal to its plane
Figure 21.93
making an angle of 900 with the direction of
30 I magnetic induction, the necessary torque is
(A) , directed up the plane of the paper (A) 1500 Nm (B) 9.4 x 10-3 Nm
2
(C) 15 Nm (D) 150 Nm
0 I
(B) , directed into the plane of the paper
6
0 I
(C) , directed towards you
6
0 I  3 1 
(D)    towards you
2   3 

Q.35 The net magnetic field at C due to the


current carrying loop is
0 I
(A) Zero (B)

0 I 0 I 30 I
(C) (D) B    ,
9 6a 2a
directed into the plane of the paper

Comprehension-2
A current carrying coil behave like short
magnet whose magnetic dipole moment
M=nIA. Where direction of M is taking along
the direction of magnetic fields on its axis and
n is no of turns A is area of coil and I is current
flowing through coil. When such a coil is put in
magnetic field (B) magnetic torque  acts on 
it as   MxB and potential energy of the
current loop in the magnetic field is u=-M.B.

Q.36 A current of 3A is flowing in a plane


circular coil of radius 1cm and having 20 turns.

21.7
www.plancess.com
Moving Charges and Magnetism

PREVIOUS YEARS’ QUESTIONS JEE MAIN


Q.39 Two very long straight parallel wires carry Q.42 A non-planar loop of conducting wire
steady currents I and –I respectively. The carrying a current I is placed as shown in the
distance between the wires is d. At a certain Fig. 21.96. Each
instant of time, a point charge q is at a point of the
equidistant from the two wires in the plane of straight section of
the wires. Its instantaneous velocity V is the loop is of length 2a. The magnetic field due
perpendicular to this plane. The magnitude of to
the force due to the magnetic field acting on this loop at
the charge at this instant is (1998) the point P(a,0,a) points in the direction(2001)
0 Iqv 0 Iqv (a)
(a) (b)
2d d
20 Iqv
(c) (d) Zero
d

Q.40 An infinitely long conductor PQR is bent


to form a right angle as shown in Fig. 21.94. A
current I flows through PQR. The magnetic field
due to this current at the point M is H1. Now,
another infinitely long straight conductor QS is
connected at Q,
so that current is
Figure 21.96
I/2 in QR as well
1 (b) 1 ˆj  kˆ  ˆi
as in QS, the
2
 ˆj  kˆ  3
 
current in PQ
remaining (C) 1 ˆi  ˆj  kˆ
  (d) 1 ˆi  kˆ
 
uncharged. The 3 2
magnetic field at
Q.43 A coil having N turns is wound tightly in
M is now H2. The
Figure 21.94 the form of a spiral with inner and outer radii a
ratio H1/H2 is
and b respectively. When a current I passes
given by (2000)
through the coil, the magnetic field at the
(a) 1/2 (b) 1
center is (2001)
(c) 2/3 (d) 2
 0NI 20NI
(a) (b)
Q.41 Two long parallel wire are at a distance 2d b a
apart. They carry steady equal currents flowing 0NI 0 IN  b
b n 
out of the plane of the paper as shown. The (c) n (d)
2(b  a) a 2(b  a)  a 
variation of the magnetic field B along the line
XX’ is given by (2000)
Q.44 Two particles A and B of masses mA and
mB respectively and having the same charge
are moving in a plane. A uniform magnetic field
exists perpendicular to this plane. The speeds

Figure 21.95 Figure 21.97

21.8
www.plancess.com
Moving Charges and Magnetism

of the particles are VA and VB respectively and (a) E  0;B  bjˆ  ckˆ ˆ B  ckˆ  aiˆ
(b) E  ai;
the trajectories are as shown in the Fig. 21.97.
(c) E  0;B  cjˆ  bkˆ ˆ B  ckˆ  bjˆ
(d) E  ai;
Then
(2001) Q.48 A current carrying loop
Figure 21. 101
(a) mA v A  mB vB is placed in a uniform
magnetic field in four different orientations, I, II,
(b) mA v A  mB vB III and IV, arrange them in the decreasing order
(c) mA  mB and v A  vB of potential energy (2003)

(d) mA  mB and v A  vB
Q.45 A long straight wire along the z-axis
carries a current I in the negative z-direction.
The magnetic vector field B at a point having
coordinate (x,y) on the z=0 plane is (2002)

(a)

0 I yiˆ  xjˆ  (b)

0 I xiˆ  yjˆ 
2 2 2 2
2(x  y ) 2(x  y )

(c)

0 I xjˆ  yiˆ  (d)

0 I xiˆ  yjˆ  Figure 21.99
2
2(x  y ) 2 2
2(x  y ) 2 (a) I > III > II > IV (b) I > II > III > IV
(c)I > IV > II > III (d) III > IV > I > II
Q.46 A particle of mass m and charge q moves
with a constant velocity v along the positive x- Q.49 An electron moving with a speed u along
direction. It enters a region containing a the position x-axis at y=0 enters a region of
uniform field B directed along the negative z- uniform magnetic field B  B0kˆ which exists
direction, extending from x=a to x=b. the to the right
minimum value of v required so that the of y-axis. The
particle can just enter the region x>b is (2002) electron
qbB q b  a  B exits from
(a) (b)
m m the region
qaB q b  a  B after
(c) (d)
m 2m sometime
with the
Q.47 For a positively charged particle moving in Figure 21.100
speed v at
a x-y plane initially along x-axis, there is a
coordinate y, then (2004)
sudden change in its path due to presence of
(a) v>u, y<0 (b) v=u, y>0
electric and/or magnetic fields beyond P. The
(c) v>u, y>0 (d) v=u, v<0
curved path is shown in the x-y plane and is
found to be non-circular.
Q.50 A magnetic field B  B0ˆj exists in the
Which one of the following combinations is
possible? (2003) region a<x<2a and B  B0ˆj , in the region
2a<x<3a,
where B0 is a
positive
constant. A
positive
point
charge
moving with Figure 21.101
Figure 21.98
a velocity

21.9
www.plancess.com
Moving Charges and Magnetism

v  v 0ˆi , where v0 is a positive constant,


0NI b 0NI b  a
(a) In   (b) In  
2(b  a)  a  2(b  a)  b  a 
0NI  b  0NI  b  a 
(c) In   (d) In 
2b  a  2b  b  a 

Figure 21.102
enters the magnetic field at x=a. The trajectory
of the charge in this region can be like (2007)

Q.51 Which of the field patterns given in the


Fig. 21.103 is valid for electric field as well as for
magnetic field? (2011)

Figure 21.103

Q.52 A long insulated copper wire is closely


wound as a spiral of N turns. The spiral has
inner radius a
and outer radius
b. The spiral lies
in the X-Y plane
and a steady
current I flows
through the wire.
The Z-
component of
the magnetic
field at the center Figure 21.104
of the spiral is
(2011)

21.10
www.plancess.com
Moving Charges and Magnetism

EXERCISE 1 JEE ADVANCED


Q.1 A system of long Q.6 Find the magnetic
four parallel conductors induction at point O, if the
whose sections with the current carrying wire is in
plane of the drawing lie the shape shown in the Fig. Figure 21.110
at the vertices of a 21.110.
square there flow four
Q.7 Find the magnitude of
equal currents. The
Figure 21.105 the magnetic induction B
directions of these
of a magnetic field
currents are as follows:
generated by a system of
Those marked  point away from the reader,
thin conductors along
while those marked with a dot point towards
which a current i is flowing
the reader. How is the vector of magnetic Figure 21.111
at a point A(O,R,O), that is
induction directed at the center of the square?
the center of a circular
Q.2 A long straight wire carriers a current of conductor of radius R. The ring is in the yz
10A directed along the negative y-axis as plane.
shown in Fig. 21.106. A uniform magnetic field
Q.8 A cylindrical conductor of radius R carriers
B0 of magnitude 10-6T
a current along its length. The current density
is directed parallel to
J, however, is not uniform over the cross section
the x-axis. What is the
of the conductor
resultant magnetic
but is a function
field at the following
of the radius
points?
according to
(a) x=0, z=2m ; Figure 21.106
J=br, where b is a
(b) x=2m, z=0
constant. Find an
(c) x=0 , z=-0.5m Figure 21.112
expression for the
Q.3 Find the magnetic field B.
magnetic field (a) at r1  R
at the center P
of square of (b) at distance r2  R , measured from the axis
side a shown in Figure 21.107
Fig. 21.107. Q.9 Electric charge q is uniformly distributed
over a rod of length l. The rod is placed parallel
Q.4 What is the magnitude of magnetic field at to a long wire carrying a current i. The
the center ‘O’ of separation between the rod and the wire is a.
loop of radius 2 Find the force needed to move the rod along
m made of its lengths with a uniform velocity v.
uniform wire when
a current of 1amp Q.10 An electron moving with a velocity
enters in the loop 5x106 ms1ˆj in the uniform electric field of
and taken out of it Figure 21.108 5x107 Vm1ˆj . Find the magnitude and direction
by two long wires
as shown in the Fig. 21.108. of a minimum uniform magnetic field in tesla
that will cause the electron to move undeviated
Q.5 Find the magnetic along it original path.
induction at the origin in
the Fig. 21.109 shown. Q.11 A charged particle (charge q, mass m) has
velocity v0 at origin in +x direction. In space
Figure 21.109 there is a uniform magnetic field B in –z

21.11
www.plancess.com
Moving Charges and Magnetism

direction. Find the y coordinate of the particle (c) (2a, 0).


when it crosses y axis.
Q.17 A square current
Q.12 A proton beam passes without deviation carrying loop made of thin
through a region of space where there are wire and having a mass
uniform transverse mutually perpendicular m=10g can rotate without
electric and magnetic field with E and B. Then friction with respect to the
the beam strikes a grounded target. Find the vertical axis OO I , passing
force imparted by a beam on the target if the through the center of the loop Figure 21.115
beam current is equal to I. at right angles to two opposite sides of the
Q.13 A conducting circular loop of radius r loop. The loop is placed in a uniform magnetic
carriers a constant current i. It is placed in a field with an induction B=10-1T directed at right
uniform magnetic field B0 such that B0 is angles to the plane of the drawing. A current
perpendicular to the plane of the loop. Find the I=2A is flowing in the loop. Find the period of
magnetic force acting on the loop. small oscillations that the loop performs about
its position of stable equilibrium.
Q.14 An arc of a circular
loop of radius R is kept Q.18 An infinitely
in the horizontal plane long straight wire
and a constant carries a
magnetic field B is conventional current
applied in the vertical I as shown in the Fig.
direction as shown in Figure 21.113 21.116. The
the Fig. 21.113. If the rectangular loop Figure 21.116
carries current I then find the force on the arc. carries a
conventional current I’ in the clockwise
Q.15 A rectangular loop of wire is oriented with direction. Find the net force on the rectangular
the left corner at the loop.
origin, one edge
along X-axis and the Q.19 3 Infinitely long thin wires each carrying
other edge along. Y- current i in the same direction, are in the x-y
axis as shown in the plane of a gravity free space. The central wire is
Fig.21.114. A along the y-axis while the other two are along
magnetic field is into Figure 21.114 x  d.
the page and has a (i) Find the locus of the points for which the
magnitude that is given by   y where  is magnetic field B is zero.
constant. Find the total magnetic force on the (ii) If the central wire is displaced along the Z-
loop if it carries current i. direction by a small amount & released, show
that it will execute simple harmonic motion. If
Q.16 A particle of charge +q and mass m the linear density of the wires is  , find the
moving under the influence of a uniform frequency of oscillation.
electric field E î and a magnetic field B k̂ enters
Q.20 Q charge is uniformly distributed over the
in I quadrant of a coordinate system at a point
same surface of a right
(0, a) with initial velocity v î and leaves the
circular cone of semi-
quadrant at a point (2a, 0) with velocity 2v ˆj . vertical angle  and
Find Magnitude of electric field height h. The cone is
(a) Rate of work done by the electric field at uniformly rotated about
point its axis at angular velocity Figure 21.117
(b) (0, a) Rate of work done by both the fields  . Calculated associated
at. magnetic dipole moment.

21.12
www.plancess.com
Moving Charges and Magnetism

Q.21 Four long wires each carrying current I as induction B=0.10 T. The coefficient of static
shown in the Fig. 21.118 are placed at the point friction between the conductor and the plane is
A, B, C and D. Find 0.1. A current of I=10A flows through the
the magnitude conductor inside the plane of this paper as
and direction of shown. What is the force that should be applied
(i) Magnetic field parallel to the inclined plane for sustaining the
at the center of the conductor at rest?
square. Figure 21.118 Q.26 An electron gun G emits electron of
(ii) Force per metre
energy 2kev traveling in the (+) ve x-direction.
acting on wire at point D.
The electron are required to hit the spot S
Q.22 A wire loop carrying current I is placed in where GS=0.1m & line GS makes an angle of
the X-Y plane as shown in the Fig. 21.119. 600 with the x-axis, as shown in the figure. A
uniform magnetic field B parallel to GS exists in
the region
outside to the
electron gun.
Find the
minimum value
of B needed to
Figure 21.119 Figure 21.121
make the
(a) If a particle with charge +Q and mass m is electron hit S.
placed at the center P and given a velocity
Q.27 Two coils each of 100 turns are held such
along NP (see Fig. 21.119). Find its
that one lies in the vertical plane with their
instantaneous acceleration.
centers coinciding. The radius of the vertical
(b) If an external uniform magnetic induction
coil is 20cm and that of the horizontal coil is
field B  Biˆ is applied, find the torque acting on
30cm. How would you neutralize the magnetic
the loop due to the field.
field of the earth at their common center? What
Q.23 (a) A rigid circular loop of radius r & mass is the current to be passed through each coil?
m lies in the xy plane on a flat table and has a Horizontal component of earth’s magnetic
current I flowing in it. At this particular place, induction = 3.49 x 10-5T and angle of dip=300.
the earth’s magnetic field is B  B ˆi  B ˆj . How
x y Q.28 An infinite wire, placed along z-axis, has
large must I be before one edge of the loop will current I1 in positive z-direction. A conducting
lift from table? rod placed in xy plane parallel to y-axis has
(b) Repeat if, B  Bx ˆi  Bzk.
ˆ current I2 in positive y-direction. The ends of
the rod subtend +300 and -600 at the origin
Q.24 A conductor with positive x-direction. The rod is at a
carrying a current is distance a from the origin. Find net force on the
placed parallel a rod.
current per unit
Q.29 A square loop of wire of edge a carries a
width j0 and width d,
current i.
as shown in the Fig.
Figure 21.120 (a) Show that B for a point on the axis of the
21.120.
loop and a distance x from its center is given
Find the force per unit length on the conductor.
40ia2
by, B 
Q.25 The figure shows a conductor of weight
  
1/2
 4x2  a2 4x2  2a2
1.0N and length L= 0.5m placed on a rough
inclined plane making an angle 300 with the (b) Can the result of the above problem be
horizontal so that conductor is perpendicular reduced to give field at x=0?
to a uniform horizontal magnetic field of

21.13
www.plancess.com
Moving Charges and Magnetism

(c) Does the square loop behave like a dipole


for points such that x>>a? If so, what is its
dipole moment?

Q.30 A straight segment OC (of length L meter)


of a circuit carrying a current I amp is placed
along the x-axis. Two infinitely line straight
wires A and B, each extending from
z   to   , are fixed by y=−a meter and
y=+a meter respectively, as shown in the Fig.
21.122. If the wires A and B each carry a current
I amp into plane of the paper. Obtain the
expression for the force acting on the segment
OC. What will be the force OC if current in the
wire B is reversed?

Figure 21.122

21.14
www.plancess.com
Moving Charges and Magnetism

EXERCISE 2 JEE ADVANCED


Q.1 Two very long straight parallel wires, 13 E0 16B0
parallel to -y direction, respectively. The wire (A) (B)
2 B0 E0
are passes through the x-axis at the point
(C) 25 (D) 5
(d, 0, 0) and (-d, 0, 0) respectively. The graph
2E0 2B0
of magnetic field z-component as one moves
along the x-axis from x=-d to x=+d, is best Q.5 A particle of specific charge (q/m) is
given by projected from the origin of coordinates with
initial velocity [ui-vj]. Uniform electric
magnetic field exist in the region along the +y
direction, of magnitude E and B. The particle
will definitely return to the origin once if
(A)  vB / 2E is an integer

 
1/2
2 2
(B) u  v B / E is an integer
Figure 21.123
(C)  vB / E in an integer
Q.2 A long thin walled (D) uB / E is an integer.
pipe of radius R carries a
current I along its length. Q.6 Two particles of charges +Q and –Q are
The current density is projected from the same point with a velocity
uniform over the v in a region of uniform magnetic field B such
circumference of the pipe. that the velocity vector makes an angle  with
The magnetic field at the the magnetic field. Their masses are M and 2M,
Figure 21.124
center of the pipe due to respectively. Then, they will meet again for the
quarter portion of the pipe shown, is first time at a point whose distance from the
0 I 2 0 I point of projection is
(A) 2
(B) (A) 2Mv cos  QB (B) 8Mv cos  QB
4 R 2R
(C) Mv cos  QB (D) 4Mv cos  QB
20 I 2
(C) (D) None
2R Q.7 A particle with charge +Q and mass m
enters a magnetic field of
Q.3 An electron (mass=9.1 x 10-31; charge=-1.6 magnitude B,
x 10-19C) experiences no deflection if subjected
existing only to the right of
to an electric field of 3.2 x 105 V/m and a the boundary YZ. The
magnetic field of 2.0 x 10-3 Wb/m2. Both the
direction of the motion of
fields are normal to the path of electron and
the particle is
to each other. If the electric field is removed,
perpendicular to the Figure 21.125
then the electron will revolve in an orbit of
direction of B. Let
radius: 2M
(A) 45m (B) 4.5m T . The time
QB
(C) 0.45m (D) 0.045m
spent by the particle in the field will be
Q.4 A particle of specific charge (A) T  (B) 2T 
(charge/mass)  starts moving from the    2     2 
(C) T   (D) T  
origin under the action of an electric field  2   2 
E  E0ˆi and magnetic field B  B0kˆ . Its velocity
Q.8 In the previous question, if the particle
   
at x0 , y 0, 0 is 4iˆ  3ˆj . The value of has-Q charge, the time spend by the particle
x0 is: in the field will be

21.15
www.plancess.com
Moving Charges and Magnetism

(A) T  (B) 2T  Q.12 A conducting ring of


   2     2  mass 2kg and radius 0.5m
(C) T   (D) T   is placed on a smooth
 2   2 
horizontal plane. The ring
Q.9 A conducting wire bent in the form of a carries a current i=4A. A Figure 21.129
2 horizontal magnetic field
parabola y  2x carriers a current i=2A as
B=10T is switched on at time t=0 as shown in
shown in Fig. Fig. 21.129. The initial angular acceleration of
21.126. This wire is the ring will be
placed in a 2 2
uniform magnetic (A) 40 rad / s (B) 20 rad / s
field B  4kˆ 2
(C) 5rad / s (D) 15 rad / s
2

Tesla. The Figure 21.126


magnetic force on MULTIPLE CORRECT QUESTION
the wire is (in newton). Q.13 In the following hexagons, made up of
(A) 16iˆ (B) 32iˆ two different material P and Q, current enters
(C) 32iˆ (D) 16iˆ and leaves from points X and Y respectively. In
which case the magnetic field at its center is
Q.10 A semicircular current carrying wire not zero.
having radius R is placed in x-y plane with its
center at origin ‘O’. There is non-uniform
magnetic field
Bo x ˆ
B k (here B o
2R
is +ve constant) is
existing in the
region. The
magnetic force
acting on Figure 21.130
semicircular wire Figure 21.127
will be along Q.14 Consider the magnetic field produced by
(A) –x-axis (B) +y-axis a finitely long current carrying wire.
(C) –y-axis (D) +x-axis (A) The lines of field will be concentric circles
with centers on the wire.
Q.11 A square loop ABCD,
(B) There can be two points in the same plane
carrying a current I, is
where magnetic fields are same.
placed near and coplanar
(C) There can be large number of points where
with a long straight
the magnetic field is same.
conductor XY carrying a
(D) The magnetic field at a point is inversely
current I, the net force on
proportional to the distance of the point from
the loop will be
the wire.
Figure 21.128
2 0 Ii  0 Ii Q.15 Current flows through uniform, square
(A) (B) frames as shown.
3 2
20 Iil 0 Iil
(C) (D)
3 2

21.16
www.plancess.com
Moving Charges and Magnetism

In which case is the magnetic field at the ASSERTION REASON QUESTION


center of the frame not zero? Q.18 Statement-1: A charged particle can
never move along a magnetic field line in
absence of any other force.
Statement-2: Force due to magnetic field is
 
given by F  q v xB .

Figure 21.131

Q.16 Which of the following statements is


correct:
(A) A charged particle enters a region of
uniform magnetic field at an angle 850 to
magnetic lines of force. The path of the particle
is a circle.
(B) An electron and proton are moving with
the same kinetic energy along the same
direction. When they pass through uniform
magnetic field perpendicular to their direction
of motion, they describe circular path.
(C) There is no change in the energy of a
charged particle moving in a magnetic field
although magnetic force acts on it.
(D) Two electrons enter with the same speed
but in opposite direction in a uniform
transverse magnetic field. Then the two
describe circle of the same radius and these
move in the same direction.

Q.17 In a region of space, a uniform magnetic


field B exists in the y-
direction. A proton is
fired from the origin,
with initial velocity v
making a small angle 
with the y-direction in
the yz plane. In the
Figure 21.132
subsequent motion of
the proton,
(A) Its X-coordinate can never be positive
(B) Its X- and z-coordinates cannot both be
zero at the same time.
(C) Its z-coordinate can never be negative.
(D) Its y-coordinate will be proportional to the
square of its time of flight.

21.17
www.plancess.com
Moving Charges and Magnetism

MATRIX MATCH TYPE QUESTION


Q.19 Two wires each carrying a steady current I are shown in four configuration in Column I. Some
of the resulting effects are described in Column II. Match the statement in Column I with the
statements in Column II and indicate your answer by darkening appropriate bubbles in the 4 x 4
matrix given in the ORS.

Column I Image Column II


(A) Point P is situated midway (P) The magnetic fields (B) at
between the wires P due to the currents in the
wires are in the same
direction.

Figure 21.133

(B) Point P is situated at the mid- (Q) The magnetic fields (B) at
point of the line joining the P due to the current in the
centers of the circular wires, wires are in opposite
which have same radii. directions.

Figure 21.134

(C) Point P is situated at the mid- (R) There is no magnetic field


point of the line joining the at P.
centers of the circular wires,
which have same radii.

Figure 21.135

Q.20 Six point charges, each of the same magnitude q, are arranged in different manners as shown
in Column II. In each case, a point M and a line PQ passing through M are shown. Let E be the
electric field and V be the electric potential at M (potential at infinity is zero) due to the given
charge distribution when it is at rest. Now, the whole system is set into rotation with a constant
angular velocity about the line PQ. Let B be the magnetic field at M and  be the magnetic moment
of the system in this condition. Assume each rotating charge to be equivalent to a steady current.

Column-I Image Column-II


(A) E=0 Charges are at the corners of a
regular hexagon. M is the center
of the hexagon. PQ is
perpendicular to the plane of
the hexagon.

Figure 21.136

21.18
www.plancess.com
Moving Charges and Magnetism

(B) V  0 Charges are on a line


perpendicular to PQ at equal
intervals. M is the midpoint
between the two innermost
charges.

Figure 21.137

(C) B=0 Charges are placed at the


corners of a rectangle of sides a
and 2a and at the mid points of
the longer sides. M is at the
center of the rectangle. PQ is
parallel to the longer sides.

Figure 21.138

(D)   0 Charges are placed at the


corners of a rectangle of sides a
and 2a and at the mid points of
the longer sides. M is at the
center of the rectangle. PQ is
parallel to the longer sides.
Figure 21.139

21.19
www.plancess.com
Moving Charges and Magnetism

PREVIOUS YEARS’ QUESTIONS JEE ADVANCED


ASSERTION AND REASON 
Q.23. A superconductor has TC 0  100K .
Q. 21. Statement I The sensitivity of a moving
When a magnetic field of 7.5 Tesla is applied,
coli galvanometer is increased by placing a
suitable magnetic material as a core inside the its TC decreases to 75K. For this material one
coil. [2008] can definitely say that when (Note: T=Tesla)a)
Statement II Soft iron has a high magnetic B  5T,TC B  80K
permeability and cannot be easily magnetized
or demagnetized. 
(b) B  5T,75K  TC B  100K

Passage Based Problem (c) B  10T,75K  TC B   100K

(d) B  10T,TC B   70K


Electrical resistance of certain material, known
as superconductors, changes abruptly from a
non-zero value to zero as their temperature is OBJECTIVE QUESTIONS II

lowered below a critical temperature TC 0 . An [ONE OR MORE CORRECT OPTION]
Q.24 A proton moving with a constant velocity
interesting property of superconductors is that
passes through a region of space without any
their critical temperature becomes smaller than
change in its velocity. If E and B represent the
TC  0  if they are placed in a magnetic field i.e., electric and magnetic fields respectively. Then,
the critical this region of space may have [1985]

temperature TC B is (a) E=0, B=0
(c) E≠0, B=0
(b) E=0, B≠0
(d) E≠0, B≠0
a function of the
magnetic strength B. Q.25. A particle of
The dependence of charge +q and
TC B  on B is shown mass m moving
under the
in the Fig. 21.141. Figure 21.141 influence of a
Q.22. In the graphs below, the resistance R of a uniform electric
superconductor is shown as a function of its field E ˆi and
temperature T for two different magnetic fields uniform
Figure 21.142
B1 (solid line) and B2 (dashed line). If B2 is magnetic field Bkˆ
follows a
larger than B1 , which of the following graphs trajectory from P to Q as shown in Fig. 21.142.
shows the correct variation of R with T in these The velocities at P and Q are viˆ and  2ˆj . Which
fields? [2010] of the following statement (s) is/are correct?
[1991]
3  mv 
2
(a) E   
4  qa 
(b) Rate of work done by the electric field at P
3  mv 2 
is  
4  a 
(c) Rate of work done by the electric field at P
is zero
Figure 21.140 (d) Rate of work done by both the fields at Q is
zero

21.20
www.plancess.com
Moving Charges and Magnetism

  2 (c)Path length of the particle in Region II is


Q.26 H ,He and O all having the same kinetic
qlB
energy pass through a region in which there is maximum when velocity v=
m
a uniform magnetic field perpendicular to their
(d) Time spent in Region II is same for any
  2
velocity. The masses of H ,He and O are 1 velocity v as long as the particle returns to
amu, 4 amu and 16 amu respectively. Then Region I
[1994]
Q.29 An electron and a proton are moving on
(a) H will, be deflected most

straight parallel paths with same velocity. They
(b) O2 will be deflected most
enter a semi-infinite region of uniform
 2
(c) He and O will be deflected equally magnetic field perpendicular to the velocity.
(d) all will be deflected equally Which of the following statement(s) is/are true?
[2011]
Q.27 Which of the following statement is (are) (a) They will never come out of the magnetic
correct in the given Fig. 21.143? [2006] field region
(b) They will come out travelling along parallel
axis
(c) They will come out at the same time
(d) They will come out at different times.

Figure 21.143
(a) Net force on the loop is zero.
(b) Net torque on the loop is zero.
(c) Loop will rotate clockwise about axis OO'
when seen from O
(d) Loop will rotate anticlockwise about OO’
when seen from O

Q.28. A particle of mass m and charge q.


moving with velocity v enters Region II normal
to the boundary as
shown in the
Fig.21.144. Region
II has a uniform
magnetic field B
perpendicular to
the plane of the
paper. The length
Figure 21.144
of the Region II is
l. Choose the correct choice (s).
[2008]
(a) The particle enters Region III only if its
qlB
velocity>
m
(b) The particle enters Region III only if its
qlB
velocity v<
m

21.21
www.plancess.com
Moving Charges and Magnetism

PLANCESSENTIAL QUESTION

EXERCISE 1 JEE MAIN/BOARDS/BOARDS


Q. 7 Q.8 Q.12 Q.20 Q.25
Q.26 Q.27

EXERCISE 2 JEE MAIN


Q.2 Q.5 Q.20 Q.22 COMPREHENSION 1 AND 2
Q.40 Q.42 Q.44 Q.45 Q.48 Q.50

EXERCISE 1 JEE ADVANCED


Q.4 Q.5 Q.16 Q.19 Q.22 Q.25
Q.30

EXERCISE 2 JEE ADVANCED


Q.1 Q.3 Q.11 Q.13 Q.15 Q.19
Q.20 Q.22

21.22
www.plancess.com
Moving Charges and Magnetism

ANSWER KEY
EXERCISE 1 JEE MAIN/BOARDS
Q.1   104 T  3.1  104 T Q.2 3.5  105 T

Q.3 4  106 T, vertically up Q.4 1.2  105 T, towards south

Q.5 0.6N m1 Q.6 4.2cm

Q.7 (i) 3.1 Nm, (b) No Q.8 5 104 T  1.6  103 T towards west

Q.9 (a) A horizontal magnetic field to magnitude 0.26T normal to the conductor in such a direction
that Fleming’s left-hand rule gives a magnetic force upward. (b) 1.176N

Q.10 (1) 1.22N m1

Q.11 (a) 2.1 N vertically downwards (b) 2.1N vertically downwards (c) 1.68N vertically downwards

Q.12 (a) 1.8  102 N m along –y direction (b) same as in (a)


(c) 1.8  10 N m along-x direction
2
(d) 1.8  102 N m at an angle of 240 with the +x direction
(e) zero (f) zero

Q.13 (a) zero (b) Zero (c) force on each electron in evB=IB(nA)= 5  1025 N .
Note: Answer (c) denotes only the magnetic force.

0 I 0 I
Q.14 B  Q.15 B 
2R 2R

Q.16 B  0nI, where n is the number of turns per unit length and I is the current flowing through
the solenoid.

Q.17 2:1 
Q.19 F  qE  q V  B 
Q.20 Circle Q.21 4  105 T

Q.22 B=14.1 Wb Q.23 1.57  102 T

7 1
Q.24 3  1012 Q.25 6  10 Nm

6
Q.26 5.9  10 N m Q.27 22.5V

Q.28 (i) 8A (ii) 3  107 T (iii) 7.68  10–6 Nm1

EXERCISE 2 JEE MAIN


Q.1 D Q.2 C Q.3 A Q.4 A Q.5 A Q.6 A
Q.7 B Q.8 D Q.9 D Q.10 B Q.11 A Q.12 B
Q.13 C Q.14 C Q.15 A Q.16 A Q.17 A Q.18 C
Q.19 D Q.20 B Q.21 B Q.22 B, D Q.23 A, B, C Q.24 A, B, C, D
Q.25 A, B Q.26 A, D Q.27 C, D Q.28 B Q.29 D Q.30 C
Q.31 D Q.32 D Q.33 (A) Q.34 (A) Q.35 D Q.36 D
Q.37 B Q.38 B

PREVIOUS YEARS’ QUESTIONS JEE MAIN

21.23
www.plancess.com
Moving Charges and Magnetism

Q.39 D Q.40 C Q.41 B Q.42 D Q.43 C Q.44 B


Q.45 A Q.46 B Q.47 B Q.48 C Q.49 D Q.50 A
Q.51 C Q.52 A

EXERCISE 1 JEE ADVANCED

Q.1 In the plane of the drawing from right to 0II'C 1 1 


left Q.18    to the left
2  a b 
6 I 0
Q.2 (a) 0 (b) 1.41  10 T, 45 in xz Q.19 z=0, x   d (ii)
3 2d 
plane, (c) 5  106 T , +x-direction
Q
Q.20 h2 tan2 
4
(1  2 2)0 I
Q.3 k̂ 0  4I 
a Q.21 (i)   along Y-axis,
Q.4 zero 4  a 
0 I  3 ˆ 1 ˆ  0  I 2  1
Q.5  k  j (ii)   10,tan4     with positive axis
4R  4   
4   2a  3
0i  3  QV 0 I  3 3 
Q.6    1 Q.22 (a)   1
4R  2 m 6a   
 
0 i  3  2ˆ
Q.7 B 
4 R

2 22  2  1  (b)   BI  
3
a j
4 

0br12 0bR3 mg
Q.8 B1  ,B2  Q.23 (a) I  (b) I  mg
3 3r2
 
1/2
rBx
r B2x  B2y
iqv
Q.9
2a 0iJ0 d ˆ
Q.10 10kˆ Q.24 tan1    k 
  2h 
2mv 0
Q.11 Q.25 0.62N<F<0.88N
qB
3
mEI Q.26 Bmin  4.7  10 T
Q.12
Be Q.27 i 1  0.1110A,i2  0.096A
Q.13 zero
0 I1 I2
Q.14 2 I R B Q.28 In (3) along –ve z direction
4

Q.15 F  a ij Q.29 (b) Yes, (c) Yes,  =Ia2.
3mv2 3 0 I 2 a2 
Q.16 (a) , (b) 3mv , (c) zero Q.30 F  ln  2  , zero
4qa 4a 2  l  a2 
m
Q.17 T0  2  0.57s
6IB

21.24
www.plancess.com
Moving Charges and Magnetism

EXERCISE 2 JEE ADVANCED


Q.1 C Q.2 A Q.3 C Q.4 C Q.5 C Q.6 D
Q.7 C Q.8 D Q.9 B Q.10 A Q.11 A Q.12 A
Q.13 A Q.14 A, B, C Q.15 C Q.16 B, C Q.17 A Q.18 D
Q.19 (A) Q, R; (B) P; (C) Q, R; (D) Q, OR; (A) Q, R; (B) P; (C) Q, R; (D) Q, S
Q.20 (A) P, R, S; (B) R, S; (C) P, Q; (D) R,S

PREVIOUS YEARS’ QUESTIONS JEE ADVANCED


Assertion and Reason
Q.21. (c)
Passage Based Problem
Q.22. (c) Q.23. (b)
Objective Questions Ii
Q.24. (a, b, d) Q.25. (a, b, d) Q.26. (a, c) Q.27. (a, c) Q.28. (a, c, d) Q.29. (b, d)

21.25
www.plancess.com
Moving Charges and Magnetism

SOLUTIONS
EXERCISE – 1 JEE MAIN 0i1i2 0  (300)2
Sol.1 B = F= = = 1.2 Nm–1
2d 3
 0NI 0  100  0.4 2   102
= = 3.1 × 10–4 T 2
2R 2  8  10 2
Since current is in.
0 I 0  35 Sol.11 B = 1.5T; r = 0.1 m
Sol.2 B = = = 3.5 × 10–5 T
2 r 1 B = 1.5 T
2 
5
7A
0 I 0  50
Sol.3 B = k̂ = k̂ = 4 × 10–6 T,
2r 5
2 
2 (a) F = i  B
vertically upward. = 7 × 0.2 ×1.5 = 2.1 N vertically downwards.
(b)
0  90 0
Sol.4 B = =  30 = 1.2 × 10–5 T, F = i 1 B
3 
2  =i
2 1Bsin  
towards south.  20 
=iB×  
Sol.5 F = I ×B  100 
1 = 7 × 0.2 × 1.5 = 2.1 N vertically downwards.
= 8 × 1 × 0.15 × = 0.6 Nm–1
2 (c)
Effective length of wire
Sol.6 F = qv  B is 16 cm in the magnetic
10 6
Since force is always perpendicular to velocity field so 8 8
so path will be a circle F = i B
mv 9.1  10 31  4.8  106 = 7 × 0.16 ×1.5
R= = = 4.2 cm
qB 1.6  10 19  6.5  10 4 = 1.68 N downwards

Sol.7 (a)  = M × B; M = INA Sol.12 Length of wire = N × 2  R


2
 = 6 × 30 ×  (0.08) ×1 × sin60º = 3.1 Nm N  2R
Final no. of turns = = 2N
(b) No R
2
2
0  20  16 0  25  18
Sol.8 B = + Magnetic moment  = INA
16 10
2 2 1 N1 A1 N  R 2 2
100 100 = = =
= 5  × 10–4 T toward west 2 N2 A2 R 
2 1
2N    
Sol.9 For tension to be zero 2
(a) F = I ×B = mg Sol.13 N = 20
60 r = 0.1 m
= 5 ×0.45 × B =  10
1000 B = 0.1 T
0.6 I=5A
B= = 0.26 T
5  0.45 (a)  = M × B = MB sin0º = 0
(b) By force equilibrium (b)F = i  B
2T = Mg + F
F = Total force is zero as is zero for a closed
= 0.06 × 9.8 + 0.06 × 9.8 = 1.176 N
loop
Sol.10 I = 300A; Force per unit length = F (c)Force on each electron = q v  B

S 21.1
www.plancess.com
Moving Charges and Magnetism

IB Sol.25 Force per unit length


= eVB = = 5 × 10–25 N
nA 0i2
F= = 6 × 10–7 N/m
Sol.14 Refer page-11 to 14 2d

Sol.15 Refer page-11, 12 Sol.26 Magnetic field inside the solenoid is


400
Sol.16 B at the centre is B =  0NI B =  0NI =  0 × ×3
0.4
N - no. of turns = 4  ×10–7 ×3 ×1000 = 12  × 10–4 T
I - current Torque on the coil is  = M × B = MB
mv = 0.4 × 10 ×  (0.01)2 × 12  × 10–4
Sol.17 R = =5.9 ×10–6 Nm
qB
RqB Sol.27 Let the resistance of the voltmeter be R
v=
m Voltage across 300  = 60 – 30 V= 30 V
vp RqB md 30
= 2 I= = 0.1 A
vd mp RqB 300
Let equivalent resistance of voltmeter and
Sol.18 Refer page:- 25-26 400Ω be Req
IReq = 30 V
Sol.19 F = q E + q v × B 0.10 Req = 30 V
Sol.20 Its path will be a circle. R  400
Req = 300  =
R  400
Sol.21 Magnetic field due to side BC is BBC
3R + 1200 = 4R
 I  3 3 3 30 I Resistance of voltmeter = R = 1200 
BBC = 0    ; BBC =
4R  2 2  4 R When voltmeter is connected to 300 
Magnetic field due to all sides will be equal so 1200  300
Req = = 240 
1500
3 30 I
Bnet = 60 6
4 R i= A A
640 64
Sol.22 Electron moving in a circle will act like a 6
Voltage measured = ×240 = 22.5 V
loop carrying current I. 64
q q qv
So, I = = = Sol.28 (i) i = 9.6A
t 2 2R 2m
 I
So magnetic field at centre = B = 0 = 0 2
 qv ⊗ ⊙
2R 4 R 10
m
Thus 11
9  109  1.6  1019  2.2  106 10
B  14.1 Wb B at m from wire B is
0.5  10 
2
10 11
  9.6 0 I
B= 0  0
Sol.23 B =  0 ni  12  10
2   2 
5000  11  11
= 4  × 10–7 × × 5 = 1.57 ×10–2 T
2 9.6  10
I= = 8A
12
Sol.24 B = 2.5 T
 0i1i2
v = 1.5 × 107 m/s (iii) Force per unit length F =
2  2
71
F = qv  B = q × 1.5 × 10 × 2.5 × 0  9.6  8
2 = = 7.68 × 10–6 Nm-1
2.5 4
= 1.6 × 10–19 ×1.5 ×107 × × 3 × 10–12
2

S 21.2
www.plancess.com
Moving Charges and Magnetism

EXERCISE – 2 JEE MAIN Sol.7 E  K1ˆj ; K1 is some constant

Sol.1 Total magnetic field at point O V  K 2ˆi


 I 3 0 I 1 F = q V  B  qE = 0
=B= 0  ·
2R' 4 2R 4  V  B = E  B = k̂
 I  3 1
= 0   
8  R' R  Sol.8 Final velocity of the particle
2
 qEt 
=v= v 20   = 2v0
Sol.2  m 
 2
 qEt  2
R v 20   = 4 v0
 m 
2
 qEt  2 3 mv 0
 I   ˆ   = 3v 0  t=
Magnetic field B = 0   (k)  m  qE
2R  2 

0 q V  r  qV sin  Sol.9
Sol.3 B = = 0
4 r 3 4 r2 •
qE
 2  100  sin30º
= 0
4 4
V0
7
= 10  25
When electric field is applied
2
Sol.4 Magnetic field at the common centre is mv 0
= qE
0 I ˆ 0 I ˆ 0 I ˆ R1
i j k
2R 2R 2R 2
mv 0
0 I R1 =
= 3 qE
2R When magnetic field is applied
Sol.5 Magnitude of magnetic field at the centre mv 0
R2 =
  20  16 0  25  18 qB
=+ 0 
2  16  10 2 2  10  10 2 R1 mv 20 qB v0 B
= =
= – 0  10 + 0  2250
3 R2 qEmv 0 E
0 4
=  0 × 1250 =  5000  = 5  10 T Sol.10 B0
4
y
Sol.6 (d, 0, 0)
a V

x
For the particle to not hit y-z plane radius of the
particle should be less than equal to d
z mv
R= ≤d
0i ˆ 0i ˆ qB0
Magnetic field = B = – i j
2a 2a qB0d
vmax =
m

S 21.3
www.plancess.com
Moving Charges and Magnetism

kq1 q2 2
kq Sol.16 Force acting on particle = q. v × B
Sol.11 Electric force Fe= =
r 2
r2  q.2 î × B = –2 ˆj
 qv   B is in +ve z direction ( k̂ )
Magnetic force = qv  0  2 
Electric force on the particle is zero.
 4 r 
  1 So when v3 =2 k̂ , force is zero.
Fm = q2v2  0  2
 4  r Sol.17 Magnetic field is in (– k̂ ) direction
Fe 2 So direction of force
k 1 c
= = 2 = 2 F = q v ×B
Fm   v 0 0 v
v 2  0  F̂ = –[– î ×(– k̂ )] = ˆj
 4 
F
mv 2mKE Sol.18
Sol.12 R = =
qB qB
2KE  1  mp
RH  (mp = mass of proton) mgcos 
eB
mgsin 
2KE  4  mp F = qVB
RHe 
eB Particle will leave the inclined plane when
F = mgcos 
2KE  16  mp
R O2   qvB = mgcos 
2eB mgcos 
So, R  = R 2 v=
He O qB
Time taken to reach v is t
mv 2mKE v = gsin  t
Sol.13 R = =
qB qB v mgcot  mcot 
t= = =
2m(2KE) 2 gsin qgB qB
R' = =R
q(3B) 9
Sol.19
y
Sol.14
Vy = 4 ⊗

Vx = 5
B =10 x
Y-component of velocity will make the particle T T

to move in circle whereas x-component of


velocity will make particle move along x-axis.
So motion is helical.

F = I d B
Sol.15 Force on a particle moving in magnetic = I (2r î )× (–0.2 k̂ ) = 20 ˆj
field is qv  B . Magnetic force is in +ve y direction
(4iˆ  3ˆj)  10 13 =1.6 × 10–19 × 2.5 ×107 (K  B) So balancing force on semi-circular ring we get
Force will be zero if direction of magnetic field 2T = 20
and velocity is same. T = 10N
So B = (0.6 î × –0.8 ˆj ) B
 (4 î + 3 ˆj ) = 1.6 × 25 ( k̂ × (0.6 î – 0.8 ˆj ) B
 B = –0.075 î + 0.1 ˆj

S 21.4
www.plancess.com
Moving Charges and Magnetism

 0I  0I
Sol.20  (A) B = –
=0
 2a 2a
B
B (B) On y-axis say at (y, 0, 0)
 0 I ˆ  0I ˆ
B= k k
2(a  y) 2(a  y)
B
So except at origin, B has only z-components
 (C) z
B B1
mgsin
Torque due to magnetic field will be balanced Bnet
by gravity.
B2
mg sin  R = I×  R2 × B sin  1
mg y
B= O
iR
2
Sol.21 Magnetic field = I × A x

Q.r 2
M
t (D) B cannot has x-component as B is
2 2r perpendicular to direction of I.
t= =
 v
Q.r 2 v Qrv Sol.25 This can be done by applying magnetic
M= =
2r 2 field in y-axis or z-axis.
y y
y

Sol.22
A(0,1,0)
x
(0,1,1)B D(1,1,1) x
x
z z
C(1,0,1)
z V
Sol.26
Magnetic field 30º
0 I 0 I B
at A = BA = ; BB = V
2  1 2 2
60º
0 I 0 I
BC = ; BD =
2  1 2  2 T1
2m
Time period T = ; a= =1
1 qB T2
Sol.23 = c2 mv sin 
0 0 radii R =
qB
So dimension of y is m/s
E R1 sin(30º ) 1
v = when E and B are both perpendicular = =
B R2 sin(60º ) 3
and perpendicular to velocity pitch = v cos  t
So dimension of x m/s P1 v cos(30º )
Dimension of RC = sec = = 3
P2 v cos(60º )
So Z= has dimension m/s abc = 1; a = bc
CR
So x, y, z have same dimensions. Sol.27 If velocity is zero, then magnetic force is
Sol.24 On x-axis zero. Energy cannot increase in magnetic field
as work done by magnetic force is zero.

S 21.5
www.plancess.com
Moving Charges and Magnetism

F = q v  B ; So force is perpendicular to its Sol.34


velocity. a

Sol.28
60º
B
60º

Magnetic field at any point is in tangential


direction. So it is not possible for a particle to
move in tangential direction by the action of 0 I 30 I
B= (sin60º  sin60º ) =
magnetic force. a 2a
4
2
Sol.29 It's velocity vector must be perpendicular
to both magnetic field and electric field. Sol.35 Net magnetic field at C is
0 I 30 I

Sol.30 F = I d  B B=–
6a
+
2a
So force acting is attractive
Sol.36 I = 3A
1 2 r = 0.04 m
i1 i2 N = 20
B = 0.5 T
d
•P
Dipole moment M = INA = 3× 20 ×  (0.01)2
r = 1.88 × 10–2 Am2

Consider a point P in space between two wires 1


Sol.37 PE = –1.88 ×10–2 ×
at a distance r from one wire. The magnetic 2
force due to wire 1 is in positive z-axis direction = –9.4 mJ
whereas due to wire 2 is in negative z-axis
direction.

Sol.31 Statement 1 is false as Ampere's circuital


law holds good for a closed path of any size and
shape around a current carrying conductor only
if the relation is independent of distance.

Sol.32

F =q v  B


mgsin  mgcos 
mg

Since angular acceleration of the mass will not


change so time period will also remain the
same.

Sol.33 Magnetic field due to curved part is


 I  2   I
B = 0  = 0
4a  3  6a

S 21.6
www.plancess.com
Moving Charges and Magnetism

PREVIOUS YEARS’ QUESTIONS JEE MAIN


Sol.39 Net magnetic field due to both the wires Magnetic field in region BX’ will be upwards
will be downward as shown in the figure. (+ve) because all points lying in this region are
 to the right of both the wires.
v
X X’
z C B
y A
×
I –I Magnetic field in region AC will be upwards

x (+ve), because points are closer to A, compared
B to B. Similarly magnetic field in region BC will
d be downwards (–ve).
 
Since, angle between v and B is 180°. Graph (b) satisfies all these conditions.
Therefore, magnetic force Therefore, correct answer is (b).
  
Fm = q ( v × B ) = 0 Sol.42 The magnetic field at P(a, 0, a) due to the
loop is equal to the vector sum of the magnetic
Sol.40 H1 = Magnetic field at M Due to PQ + fields produced by loops ABCDA and AFEBA as
magnetic field at M due to QR shown in the figure.
But magnetic field at M due to QR = 0 C
 Magnetic field at M due to PQ (or due to
current I in PQ) = H1 P(a, 0, a)
Now H2 = Magnetic field at M due to PQ

(current I) + magnetic field at M due to
D B î
QS (current I/2) E
+ magnetic field at M due to QR
ˆj
H1 3 H1 2
= H1 + + 0 = H1 ; =
2 2 H2 3
Note: Magnetic field at any point lying on the F
current carrying straight conductor is zero.
A
B=0 Magnetic field due to loop ABCDA will be along
î and due to loop AFE BA, along k̂ . Magnitude
of magnetic field due to both the loops will be
equal. Therefore, direction of resultant
i 1
magnetic field at P will be ( î + k̂ ).
2
Note: This is a common practice, when by
assuming equal currents in opposite directions
Sol.41 If the current flows out of the paper, the in an imaginary wire (here AB) loops are
magnetic field at points to the right of the wire completed and solution becomes easy.
will be upwards and to the left will be
Sol.43 Consider an element of thickness dr at a
downwards as shown in figure.
 distance r from the centre. The number of turns
B  N 
i in this element, dN =   dr

b – a
B Magnetic field due to this element at the centre

B of the coil will be
 (dN)I  I N dr
Now, let us come to the problem. dB = 0 = 0 .
Magnetic field at C = 0 2r 2 b–a r
r b
 0NI b
B=  dB =
r a
ln  
2(b – a)  a 

S 21.7
www.plancess.com
Moving Charges and Magnetism

comes in k̂ direction which is not acceptable as


the particle is moving in x-y plane. Only in
b option (b) the particle can move in x-y plane.
dr
   
r
a In option (d) F net = q E + q ( v × B )
Initial velocity is along x-direction. So, let

v =v î

Note : The idea of this question is taken from F net = qa î + q [(v î ) × (c k̂ + b ˆj )]
question number 3.245 of IE Irodov.
= qa î – qvc ˆj + qvb k̂
mv 
Sol.44 Radius of the circle = In option (b) F net = q (a î ) + q [(v î ) ×
Bq
or radius  mv if B and q are same. (c k̂ + a î )] = qa î – qvc ˆj
(Radius)A > (Radius)B  mAvA > mBvB  
 Sol.48 U = – MB = – MB cos 
Sol.45 Magnetic field at P is B , perpendicular 
to OP in the direction shown in figure. Here, M = magnetic moment of the loop
 
y  = angle between M and B
P(x, y) U is maximum when  = 180° and minimum when
r  = 0°. So, as decreases from 180° to 0° its PE
 
i  B also decrease.
× x
o
Sol.49 Magnetic force does not change the
speed of charged particle. Hence, v = u. Further
magnetic field on the electron in the given
 condition is along negative y-axis in the
So, B = B sin  î – B cos  ˆj starting. Or it describes a circular path in
 I clockwise direction. Hence, when it exits from
Here, B = 0
2 r the field, y < 0.
y x Therefore, the correct option is (d)
sin  = and cos  =
r r   
Sol.50 F m = q ( v × B )
 I 1  I( y î – xˆj)
 B = 0 . 2 (y î – x ˆj ) = 0 2 Correct option is (a)
2 r 2( x  y 2 )
Sol.51 Correct answer is (c), because induced
(as r2 = x2 + y2)
electric field lines (produced by change in
Sol.46 If (b – a)  r magnetic field) and magnetic field lines form
(r = radius of circular path of particle) closed loops.
The particle cannot enter the region x > b.
Sol.52 If we take a small strip of dr at distance r
So, to enter in the region x > b
mv q (b – a)B from centre, then number of turns in this strip
r > (b – a) or > (b – a) or v >  N 
Bq m would be, dN =   dr
b – a
Sol.47 Electric field can deviate the path of the Magnetic field due to this element at the centre
particle in the shown direction only when it is of the coil will be
along negative y-direction. In the given options  (dN)I  NI dr
 dB = 0 = 0
E is either zero or along x-direction. Hence, it 2r (b – a) r
is the magnetic field which is really responsible r b
 0NI b
for its curved path. Options (a) and (c) cannot B=  dB =
r a
2(b – a)
in  
a
(A).
be accepted as the path will be circular in that
case. Option (d) is wrong because in that case
component of net force on the particle also

S 21.8
www.plancess.com
Moving Charges and Magnetism

I1 I2
EXERCISE – 1 JEE ADVANCED =–
0 I
× 2 ×4 k̂
1 ⊗ F2+F4
⊗2
2a
Sol.1 Fres
45º
I1 = I2 = I3 = I4 2 20 I 45º
=  k̂ F1+F3
 F1 = F 2 = F 3 = F 4 = F a
 2F Magnetic field due to 4 ⊙ ⊙3
2F 0 Ikˆ 0 I
infinite length wire = Bw = = k̂
45º a a
2 2 F 2  
45º 2
2F (1  2 2)0 I
Net magnetic field = k̂
Resultant force will be 2 2 F from right to left a

Sol.2 Let magnetic field due to wire be Bw y


(a) x = 0, z = 2m; Sol.4 2
45º 1
 I ˆi x
B = B0 + Bw = – 0 + 10–6 î 1amp 1
2  2
z
= –10–7 ×10 î + 10–6 î i2
=0 
(b) x = 2m, z = 0 1
0 I i1 = 2  1 = amp
B = B0 + Bw = k̂ + 10–6 î 2 4
2  2 1 3
i2 = 1 – = amp
B = 10–6 k̂ + 10–6 î = 2 ×10–6 T 4 4
(c) x=0, z=–0.5m 1
0    3 
B= B0 + Bw  4   2  k̂
Magnetic field due to i1 = B1 =–
  10 2 2  2 
= 10–6 î + 0  
1  
2 
2 30
= k̂
= 10–6 î + 4 × 10–7 × 10 î 8 2 4
= 5 ×10–6 î T 3 
0    
Sol.3 Magnetic field can be found as the super Magnetic field due to i2 = B2 =  4   2  k̂
position of both given below. 2 2 2
I 30 1
y 45º =  k̂
45º 8 2 4
a/2
Magnetic field due to wire in x-direction = B3
x  1
B3 = 0 (sin(–45º )  sin90º ) kˆ
z
4  1
a 1 1m
1 2
Magnetic field due to loop = B 0  1 ˆ 1
B3 =  1   k 1amp
  4  2
 
 0 I  1 1  ˆ Magnetic field due to wire in negative y-
–     4k
  a   2 2 
direction =By
 4     1 
  2   By = –  0 (sin(–45º )  sin90º )  kˆ
 4  1 

S 21.9
www.plancess.com
Moving Charges and Magnetism

0  1 ˆ
=   1   k 0
4  2
(b)
Net magnetic field = B = B1 + B2 + B3 + B4 = 0

Sol.5 Magnetic Induction


 I 1  I1 I By ampere’s law
B = 0   kˆ  0   kˆ  0 ˆj  r0 
2(2R)  4  2R  4  4 R
 I 3

B×2  r2 = 0 (JdA) = 0  br2r dr 
  
1  0 
= 0  kˆ  ˆj
4R  4   r03
B×2  r2 = 0 2b
Sol.6 Magnetic Induction 3
  0br03
0 I  3 2  ˆ 0 I ˆ B=
B=  k k 3r2
2R  2  4 R
 
 
0 I 3 ˆ 0 I ˆ Sol.9
=  k k i
2R 4 4 R a
 I  3 
= 0   1 kˆ
4R  2 

Sol.7 Magnetic Induction  I


Magnetic Force = qVB = qV  0 
 I  I
B = 0 ˆi – 0 ˆi – 0 kˆ
 I  2a 
2R 4 R 4 R y
7
0 I  I E= 5× 10
= [2  1] ˆi – 0 kˆ Sol.10
4 R 4 R x
 I  2
= 0 4   1  4  1 6
V= 5× 10 m/s z
4 R  
Magnetic force = qVB
=
0 I
4 R
 2
2 2  2  1  Electric force = qE
When both forces are equal in magnitude and
Sol.8 We will find magnetic field B by ampere’s opposite in direction then net force on charged
law. particle is zero.
qVB = qE
 B.dl   0 I IN
E 5  10
7
B= = = 10 T
V 5  106
r and direction is in positive k̂ direction
y

(a) For r1 < R Sol.11


B×2  r1 = 0   JdA 
 r1 
 
= μ0  br2r dr 
 q, m V
x
0 
3
2br1 y coordinate is equal to twice the radius of the
B×2  r1 = 0
3 circle
0br12 y = 2R
B
3

S 21.10
www.plancess.com
Moving Charges and Magnetism

mV0 B
R= xxxx
qB Sol.16 xxxx
xxxx
E î
2mV0
y=
qB
a
Sol.12 We know that velocity of charged 2a
E 2v
particle = v =
B
(a)Work done by Electric Field = Change in
mv Kinetic Energy
Force = Change in momentum per sec =
t 1 1
2 2
e ∫F.dx = m(2v)  mv
I= 2 2
t 3 2
mEI qE × 2a= mv
F= 2
B·e 3mv 2
E=
Sol.13 Force acting on a wire carrying current 4qa
3

F = I d B (b) Rate of work done = F.v = qE.v =
4a
mv 3
Since B is uniform so (c) Work done by magnetic field is always zero.
F = I ( d )  B Work done by electric field = F.vˆ
For a loop  d  0 = qE î . (–2v ˆj )= 0
So F = 0
ˆj O
Sol.17 
xB P
Sol.14 Q B
i
I R R î

k̂ S
O1 R
 2Riˆ
Force = I d  B  Consider a loop PQRS placed in uniform
Since B is constant so magnetic field B in such a way that the normal
F = I ( d )  B to coil subtends an angle  to the direction of
F=I· B B when a current I flows through the loop
F= I  2Riˆ  ( Bk)
ˆ
 clockwise.
The sides PQ and RS are perpendicular to the
= I 2RBjˆ field and equal and opposite forces of
magnitude I and B act upwards and downwards
= 2IRBjˆ
respectively. Equal and opposite forces act on
Sol.15 F = F1 + F2 + F3 + F4 sides QR and PS towards right and left of coil.
= i∫(d 1 ×B1)+ i∫ (d 2×B2) + i∫ (d 3×B3) + The resultant force is zero but resultant torque
i∫ (d 4×B4)
is not zero. The forces on sides PQ and RS
 a  a produce a torque due to a single turn which is
=  i dyjˆ  (y)( k)
ˆ   i(aiˆ  y( k))
ˆ  i dyjˆ  ( y)kˆ  I  0
    given by
 0 
 = I 2Bsin 
0

2 2
a ˆ a ˆ for small  , sin  ≈ 
F1 = i  i  i a2ˆj  i i = i a2ˆj
2 2  = I 2B ...(1)
 = I

S 21.11
www.plancess.com
Moving Charges and Magnetism

m 2  z
m 2 (ii)
=  2   2 
 4 12 4 4 
 
2 ⊗ z
 1 1
2 m 4 m 2
= m    =   ...(2)
 24 8  8 3 6 ⊗ ⊗ x
By (1) and (2) Let the middle wire is displaced by z distance in
2
2 m positive z-direction.
I Bθ =

6 Attractive force acting on wire is F
6 IB z
 =  cos   F F
2 2
m d z θ z
2 6 IB 0i2
 = F= d
m
2 d2  z2
m 10 2 Resultant force is downward
Time period = 2 = 2
6 IB 6  2  10 1
20i2 z
Fnet = –2 Fcos  = ·
1 2 d2  z2 d2  z2
= 2 = 0.57 sec
120
0i2 z
Fnet =
Sol.18 Net force acting on the loop = F  (d2  z 2 )
 I I'c 0 I I'c  I I'c  1 1  For small z
F= 0  = 0   
2a 2b 2  a b  0i2 z
This loop will experience attractive forces. Fnet = = a
 d2
y
0i2
 =
Sol.19 (i) d2
 1 0i2 i 0
F= = =
i 2 2 d 2 2d 

x 
d y
Sol.20 l cos  = h
d
y
θ
Net Force at some point x, y is h
0 I  I 0 I d
Fnet = + 0 + =0
2(x  d) 2x 2(x  d)
1 1 1
 + + =0
nd x xd h tan 
2x 1 Take a ring at distance y from the top point of
+ =0
2
x d
2 x the cone.
2
2x  x  d
2 2 Magnetic moment M = IA
= 0  3x2 = d2 dM=
x(x2  d2 )
 
d  Q 2ydy tan   
x=   ·  ·  (y tan )2
3  (htan ) h cos   2
 
Net force will be zero only in x–y plane  cos  
h
d Q tan3  Q tan  h
2 4
i.e. when z = 0 and x = 
3
= 
0 h2 tan 
· y3dy =
h2
·
4

S 21.12
www.plancess.com
Moving Charges and Magnetism

1  I 30 I   I 1 3
= Q  tan2  h2 B=  0   k̂ = 0    k̂
4  6R 2R  2R  3  
 
Sol.21 (i) BC = BA = BB = BD = B
qv  ˆ  I1 3  ˆ
y Force = qv×B =
2 
 
 i  3 ˆj  0  
2R  3
k 
  
0 I  2 BD + BB  
B= ⊙ ⊗
2 2 a 45º Qv 0 I  3 3 
BC + BA
x
=   1
m 6a   
Net magnetic ⊙ ⊗
(b) Net Torque = M  B = I AB ˆj
field is
 2 3a2  ˆ
=I  a  B j
0 I  2 3 4 
 
Bnet = B 2 = along y-axis
2 a  3 2 ˆ
= B I  a j
(ii) 3 4 
F2  
D A
F1 Sol.23 (a) Net Torque on the loop is
F1
 = – MBx ˆj + MBy î = r Bx  B y
2 2 2
...(1)
By Torque balance
C B
mgr =  ...(2)
0 I 2 By (1) and (2)
F1 =
2(2a) mg
I=
0 I 2 r B2x  B2y
F2 =
2(2 2a) (b) Net Torque is  = –MBx ˆj
2 2
F2 0 I2   1  3 0 I  |  | = Ir B x
Fx = F 1 + = 1   =
2 4a  2  8a By torque balance
2 mgr = 
F2 0 I2   1  0 I 
Fy = F 1 – = 1   = mgr = Iπr2 Bx
2 4a  2  8a
mg
I=
0 I2 2 rB x
Net force = 13 ; 1
8a
Sol.24 Magnetic field due to sheet of width d
    I2 
=  0    10 and infinite length at a distance h is given by
 4   2a 
   d 
 j
Radius = R = a B = 0 0 tan–1  2  ˆi
  h 
 
v  
R ˆ
Sol.22 (a) = j
60º 60º
F = i B
R i j 1  d  ˆ
2 F = 0 0 tan   (–k)
 2n
 
1 3 ˆ R
v = v  ˆi  j
2 2  Sol.25

  2   I
 I   I  3 3 
B =  0  3  0     k̂
 2R  2  4 R  2 2  
    
   2 

S 21.13
www.plancess.com
Moving Charges and Magnetism

Force= I B = 10 × 0.5 × 0.1 20 2  9.1  1031  2000  1.6  1019


1 =
Force = N upward on inclined plane 2  1.6  1019
2
2  9.1  1031  2000
= 10
1.6  1019
3 3 B = 10 2.275  104

  2
2 B = 4.7 × 10–3 T
1
mg 
2 1
2 mg  N Sol.27 To neutralize the magnetic field, current
2
in vertical ring should be such that the
 mgcos  magnitude of magnetic field is 3.49×10–5 T and
current in horizontal ring should be such that
the magnitude of magnetic field is
F
1
 3.49  105
3 3
3 3
4
4
3 For vertical ring
 .1  F =  NI   100  I
4 B= 0 = 0
2r 2  0.2
3 3 3
F= – 3.49×10–5 = 0  250 I
4 40
5 5
3.49  10 3.49  10
3  3 I= = = 0.111 A
Fmin = 1   = 0.62 N 0  250 4   10 7  250
4  10 
For horizontal ring
3 3 3  NI 1   100 I
F= + B= 0   3.49  105 = 0
4 40 2r 2  0.3
3
Fmax = 0.88 N
 I = 0.096 A
Sol.26
S
Sol.28
i1
0.1m
z y y
v
2 dB

dy
G v
30º
x dB
60º (a, 0 , 0) 
R y
dB
⊙ x
dB (a, 0 , 0)
v 3
2
Electron will move in helical path with pitch =
0.1 m. For minimum value of B particle should Force on dy element in x direction is
reach at point S in a single revolution.  dF =  i dy Bsin 2
2m
Time period T = Rd 0i1
qB F= i2 ·
cos  2R
sin 
v 30
so 0.1 = T 0i1i2
2 =
2  tan  d
v·2m 60
0.1 =
2·qB 0i1i2 30
F= [ n cos ]60
20mv 2
B’ =
2q

S 21.14
www.plancess.com
Moving Charges and Magnetism

0i1i2 0i1i2 Net resultant = B13 + B24


= n 3= n3
2 4 20 Ia2
= 1
Sol.29 (a)|B1| = |B2| = |B3| = |B4|  a2  2
(4x 2  a2 )  x 2  
 2 
a2 
x2 
4 (b)Yes
R

B1  B3
Sol.30
B13Q ⊗
P
x
x
2 
x
i Qa
1 3 a B
2  
4 R R dxR B
Q
P ⊗ 
a2
x2  R
2
a z
x y
a2 L
2 2
x 
4 R 
x
x a R
a
2
a2 
x x2 
2 R
Q Bres = 2Bcos 
x



 
Fres = I d  Bres
0 I  a a 
B1 =  R d 2  0 I a
a 
2 
a
2
a
2 

=  I  sin  2R
cos  ; sin  
L2  a2
4  x2  2 x2  2 x2 
4  2 2 
 0 I2 0 I2



0 I a = cot  d = ln(sin )90
B1 = ·  
2 2 
2 a 2 a 2
4 x  x 
4 2 0 I 
2
a 
= ln   1
Resultant of B1 and B3 is B13 = 2B1cos  2  L2  a2 
 
a
2  0 I a If direction of current in B is reversed then
B13 = · 2
2 2 2 resultant magnetic field will become horizontal
2 a 2 a 2 a
4 x  x  x  and so net force will be zero.
4 2 4
0 Ia2
= 1
 a2  a2  2
4   x2   x2  
 4  2 
 
0 I a2
B13 = 1
 a2  2
(4x 2  a2 )  x 2  
 2 

0 Ia2
Similarly B24 = 1
 a2  2
(4x 2  a2 )  x 2  
 2 

S 21.15
www.plancess.com
Moving Charges and Magnetism

EXERCISE – 2 JEE ADVANCED Sol.5 Particle is moving in helix along y-axis. So


the time taken by particle to reach in
Sol.1
4I x-z plane should be integral multiple of time
taken to complete one revolution.
y

(–d, 0, 0) (d, 0, 0)

Magnetic field at some x is given by x


0 (4 I) ˆ 0 I ˆ
k k
2(d  x) 2(d  x)
 I 4 1  0 I  5d  3x  z
= 0    =  
2  d  x d  x  2  d2  x2  Helical motion of the particle
It corresponds to graph (c)
2mv  2m 
Sol.2   =  n
By  qE  qB 
 d Bv
B Bx Rd n=
E
 Bv 
Magnetic field at the centre due to Rdθ So   should be an integer
component is  E 
 I  Sol.6 Both particles will move in helix. They will
0  ·Rd 
 2R  cos  = 0 I meet for the first time when mass m will

Bx = dB x = 
2R 42R complete two revolutions and mass 2m will
 I  complete one revolution. Time taken to
0  ·Rd  complete one rotation.
 2R  sin  = 0 I

By = dB y = 
2R 42R t1 =
2  2M
; t2 =
22M
qB qB
0 I
B= 1 1 Distance from the point of projection = tv cosθ
42R 4 M 4 Mv cos 
= vcos  =
E qB qB
Sol.3 V = for no deflection to occur
B
Sol.7
3.2  105
V= = 1.6 × 108 m/s
2  10 3 
VR

mv 9.1  1031  1.6  108 R 2
R= = = 0.45 m
qB 1.6  1019  2  10 3 V 
 R
q
Sol.4 =  R
m
1    2     2 
Work done by electric field = qE0x0= m(25–0)
2 Time taken =  =  T
    2 
25m 25
x0 = =
2qE0 2E0

S 21.16
www.plancess.com
Moving Charges and Magnetism

Sol.8 V
 Sol.12
R

  2R

B

R

R
V
   2     2 
Time taken =   =  T Torque on the ring due to magnetic field is
    2   = MBsin 
(2, 2)  = I ×  R2 × B = I 
Sol.9 MR 2 
I  R2 × B =
2
2  4    10
R  =
2
(2, –2) = 40  red/sec2
Magnetic force is given by I Q Y
Sol.13
dFm = i ∫d  B = i∫d(– ˆj ) × (–4 k̂ ) = 4i∫d î Q
P
since and B are perpendicular so
i1
dfm = 8∫d î = 8 × 4 î = 32 î P Q
i
Sol.10 X P i2 II
y
Let us assume that resistance of p material is ρ
and that of Q is q.
d
I 2  q 2q  
i1 = i , i2 = i

x
3(  q) 3(  q)
(–R,0,0) (R,0,0)
i1 2  q
=
i2 2q  
z We know that B  i
So B1 = magnetic field due to I part
B
F = I∫R dθ (sin  î + cos  ˆj ) × 0 (–R cos  ) k̂ B2 = magnetic field due to II part
2R
For the magnetic field to be zero B1 = –B2

IB0R should hold.
=
2  (sin  ˆi  cos  ˆj)  ( cos  k)ˆ d Bi i 2  q
0 But  1 =  –1
 Bi i2 2q  
 (sin  cos  ˆj – cos  ˆi) d
2
=
So magnetic field will not be zero at centre. In
0
(B), (C) and (D) i1 = i2 so magnetic field is zero

 sin2 ˆj (1  cos2)iˆ  at centre.
=  
0
2

2
 d


   Sol.14 i
  cos2  ˆ    ˆ  sin2  ˆ
=   j    i   i
 4 0  2 0  4 0
 
= 0 – î  0 = – î
2 2

Sol.11 Refer Q.18 Exercise-I JEE Advanced. 0 I


B= (sin 1 + sin 2 )
2 r

S 21.17
www.plancess.com
Moving Charges and Magnetism

By ampere's law magnetic field on a ring with i


By symmetry i1 = i2 =
centre as wire is same. 2
1 So magnetic field due to four sides of square
B / as 1 and 2 are also dependent on r.
r will cancel out. Magnetic field due to two
infinitely long wires will also cancel out as they
i I i are equal in magnitude and opposite in
Sol.15
i1 direction.
So net magnetic field is zero.

Sol.16 (A) Motion is helical in nature


(B) They will follow circular path with radius
i2
II 2mKE
R=
In (A) qB
3 i (C) Work done by magnetic force is always zero.
i1 = i ; i2 =
4 4
0i1 30i2 (D)
B= (sin 1  sin 2 ) + (sin 1  sin 2 ) B
l
4 4 x x x x x x x x x
2 2 x x x x x x x x x
i x x x x x x x x x
(B) V
x x x x x x x x x
i1 x x x x x x x x x
V
x x x x x x x x x
x x x x x x x x x

i2
i Sol.17 z B
v sin 
V
By symmetry i1 = i2 and magnetic field will be
cancelled out by both the parts. 
i1 y
(C) v cos 
45º 45º B
x
L i2 z

3 i
i1 = i ; i2 =
4 4 y
Let magnetic field due to sides of square be Bs
3 i x
0 i1  30  So its x coordinate cannot be positive.
4 1 1  4 1 1 
Bs =    k̂ +    Its x- and z- coordinate will be zero when
L  2 2 L
4 4  2 2
particle will complete one revolution.
2 2
Bs = 0 y - Coordinate = v cos  t
But magnetic field due to 2 infinitely long wires Sol.18 If initially velocity of charged particle is
is not zero so net magnetic field is zero. in the direction of magnetic field then force
i
(D) acting on it is zero and particle will continue to
i1
move in the same direction. So statement 1 is
false.
i Sol.19  = MB sin 90º = 9.4 ×10–3 Nm
i i2 (A) Magnetic field is in opposite direction. Since
current is in same direction so they will attract

S 21.18
www.plancess.com
Moving Charges and Magnetism

each other. Magnetic field is equal in Q


magnitude at P so magnetic field at P is zero. +• •+

(B) •
F a
B B
b M
P •–
–•

Magnetic field at P is in the same direction. •


+
Wires will attract as the current is in the same P
direction. E=0
(C) Magnetic field at P is in opposite direction Electric field will cancel out due to symmetry
due to two wires and has same magnitude. So Kq Kq
V=– ×3+ ×3≠0
net magnetic field is zero at P. Wires will attract a b
each other as current is in the same direction. B is not zero as current due to rotating charge
(D) Magnetic field will be in opposite direction is non-zero.
and wires will repel each other as current is in  = INA
opposite sense. as I ≠ 0   ≠ 0
– + –
+ –
Sol.20
F
r
F P Q

M
• +

– + –
F
+ – Electric field is zero. By symmetry electric field
Electric field is zero at point M will cancel out each other.
3Kq 3Kq Kq Kq
Electric potential =  =0 V= 4 2  0
r r  5a a
  2
Magnetic field is zero as current due to rotating  2 
 
charge is zero.
Let I be the current due to moving charge
Magnetic moment= INA = 0 × NA = 0
 I 2x0 Ia2
So B = 0 – 0
Kq Kq Kq Kq Kq Kq 2a 3
E
 5a 
2

 5a 
2

 3a 
2

 3a 
2

a
2

a
2
0
 
2 2a2 2
           
 2  2  2  2 2 2  = INA
 = 2× Ia2 – Ia2 = Ia2
P

– + – + – +
a M

Q
Kq Kq Kq Kq Kq Kq
V      0
5a 5a 3a 3a a a
2 2 2 2 2 2

B = 0 as current due to rotating charge is zero.


 = 0 as current due to rotating charge is zero.

S 21.19
www.plancess.com
Moving Charges and Magnetism

PREVIOUS YEARS’ QUESTIONS JEE ADVANCED

Assertion and Reason  3 mv 2   mv3 


= q   v= 3  
Sol.21 c = BINA 4 qa  4  a 
   
 BNA 
  I Therefore, option (b) is also correct. Rate of
 c  work done at Q:
 
Sol.22 If B2 > B1, critical temperature, (at which of electric field = F e . v = (qE)(2v)cos 90° = 0
resistance of semiconductors abruptly and of magnetic field is always zero. Therefore,
becomes zero) in case 2 will be less than option (d) is also correct.
compared to case1. 
Note that F e = qE î
 The correct option is (a).
Using iron core, value of magnetic field increases.
So, deflection increases for same current. Hence, Sol.26
sensitivity increases.
Soft iron can be easily magnetized or
demagnetized.
 Correct option is (c) mv P 2km
r= = =
Bq Bq Bq
Sol.23 With increase in temperature, TC is
m
decreasing. i.e., r 
q
TC(0) = 100 K
If K and B are same.
TC = 75 K at B = 7.5 T
Hence, at B = 5 T, TC should lie between 75 K 1 4 16
i.e., rH : rHe : rO 2  = : :
and 100 K. 1 1 2
Hence, the correct option should be (b). =1:2:3
Therefore, He+ and O2+ will be deflected
 
Sol.24 If both E and B are zero, then Fe and Fm equally but H+ having the least radius will be
deflected most.
both are zero. Hence, velocity may remain
constant. Therefore, option (a) is correct. 
Sol.27 F BA = 0, because magnetic lines are
If E = 0, B  0 but velocity is parallel or
 parallel to this wire.
antiparallel to magnetic field, then also Fe and 
F CD = 0, because magnetic lines are

Fm both are zero. Hence, option (b) is also antiparallel to this wire.

correct. F CB is perpendicular to paper outwards and
  
If E  0, B  0 but Fe + Fm = 0, then also velocity F AD is perpendicular to paper inwards. These
may remain constant or option (d) is also two forces (although calculated by integration)
correct. cancel each other but produce a torque which
tend to rotate the loop in clockwise direction
Sol.25 Magnetic force does not do work. From about an axis OO’.
work-energy theorem:
1
WFe = KE or (qE)(2a) = m[4v2 – v2]
2 X X
2
3  mv  X X
or E =   BqI
4  qa  Sol.28 v=
X X m
 Correct option is (a)
At P, rate of work done by electric field X X
 
= F e . v = (qE)(v) cos 0°

S 21.20
www.plancess.com
Moving Charges and Magnetism
 
v  B in region II. Therefore, path of particle
is circle in region II.
Particle enters in region III if, radius of circular
path, r > 
mv
or >
Bq
BqI
or v >
m
BqI mv
If v = ,r= = I, particle will turn back
m Bq
and path length will be maximum. If particle
returns to region I, time spent in region II will
be:
T m
t= = , which is independent of v.
2 Bq
 Correct options or (a), (c) and (d).
mv
Sol.29 r = or r  m
Bq
 re < rp as me < mp
2m
Further, T = or T  m
Bq
X X X

X X X
×

Te Tp
 Te < Tp, te = and tp =
2 2
or te < tp
 Correct options are (b) and (d).

S 21.21
www.plancess.com

Das könnte Ihnen auch gefallen